Why limits give us the exact value of the slope of the tangent line?












5












$begingroup$


Limits tell us how functions behave at $xto a$, not how they behave at $x = a$. However, in limits we plug $x = a$ as an approximation of $xto a$, so: why the limits give us the exact value of slope of the tangent line, despite being just an approximation of what happend around $x$?










share|cite|improve this question











$endgroup$












  • $begingroup$
    "But in limits we plug $x=a$ ..." --- This is only a technique that can sometimes be applied. See this answer.
    $endgroup$
    – Dave L. Renfro
    Apr 19 at 7:46












  • $begingroup$
    First off, we don't plug anything. That's just one heuristic technique for evaluating limits, and it works only for continuous functions. As to your question, limiting procedures give a useful way for extending concepts in a natural way (in a way that completes a class of objects) and thus help us naturally assign values to objects that don't have values in the usual sense of finite operations. In courses in analysis, it is shown that these limiting values behave essentially like their finite-procedure counterpart, and that they generalise their domain of application. These are both desirable.
    $endgroup$
    – Allawonder
    Apr 19 at 8:58










  • $begingroup$
    Related (maybe): math.stackexchange.com/questions/2491184/…
    $endgroup$
    – Hans Lundmark
    Apr 19 at 11:09










  • $begingroup$
    If you think it is just an approximation, can you give some bounds on the error this approximation has?
    $endgroup$
    – Hagen von Eitzen
    Apr 19 at 11:25










  • $begingroup$
    Possible duplicate (also unanswered): math.stackexchange.com/q/1053482/243259
    $endgroup$
    – Quelklef
    Apr 19 at 13:07
















5












$begingroup$


Limits tell us how functions behave at $xto a$, not how they behave at $x = a$. However, in limits we plug $x = a$ as an approximation of $xto a$, so: why the limits give us the exact value of slope of the tangent line, despite being just an approximation of what happend around $x$?










share|cite|improve this question











$endgroup$












  • $begingroup$
    "But in limits we plug $x=a$ ..." --- This is only a technique that can sometimes be applied. See this answer.
    $endgroup$
    – Dave L. Renfro
    Apr 19 at 7:46












  • $begingroup$
    First off, we don't plug anything. That's just one heuristic technique for evaluating limits, and it works only for continuous functions. As to your question, limiting procedures give a useful way for extending concepts in a natural way (in a way that completes a class of objects) and thus help us naturally assign values to objects that don't have values in the usual sense of finite operations. In courses in analysis, it is shown that these limiting values behave essentially like their finite-procedure counterpart, and that they generalise their domain of application. These are both desirable.
    $endgroup$
    – Allawonder
    Apr 19 at 8:58










  • $begingroup$
    Related (maybe): math.stackexchange.com/questions/2491184/…
    $endgroup$
    – Hans Lundmark
    Apr 19 at 11:09










  • $begingroup$
    If you think it is just an approximation, can you give some bounds on the error this approximation has?
    $endgroup$
    – Hagen von Eitzen
    Apr 19 at 11:25










  • $begingroup$
    Possible duplicate (also unanswered): math.stackexchange.com/q/1053482/243259
    $endgroup$
    – Quelklef
    Apr 19 at 13:07














5












5








5





$begingroup$


Limits tell us how functions behave at $xto a$, not how they behave at $x = a$. However, in limits we plug $x = a$ as an approximation of $xto a$, so: why the limits give us the exact value of slope of the tangent line, despite being just an approximation of what happend around $x$?










share|cite|improve this question











$endgroup$




Limits tell us how functions behave at $xto a$, not how they behave at $x = a$. However, in limits we plug $x = a$ as an approximation of $xto a$, so: why the limits give us the exact value of slope of the tangent line, despite being just an approximation of what happend around $x$?







limits tangent-line slope






share|cite|improve this question















share|cite|improve this question













share|cite|improve this question




share|cite|improve this question








edited Apr 19 at 8:40









BowPark

567720




567720










asked Apr 19 at 7:02









Mohammad AlshareefMohammad Alshareef

316




316












  • $begingroup$
    "But in limits we plug $x=a$ ..." --- This is only a technique that can sometimes be applied. See this answer.
    $endgroup$
    – Dave L. Renfro
    Apr 19 at 7:46












  • $begingroup$
    First off, we don't plug anything. That's just one heuristic technique for evaluating limits, and it works only for continuous functions. As to your question, limiting procedures give a useful way for extending concepts in a natural way (in a way that completes a class of objects) and thus help us naturally assign values to objects that don't have values in the usual sense of finite operations. In courses in analysis, it is shown that these limiting values behave essentially like their finite-procedure counterpart, and that they generalise their domain of application. These are both desirable.
    $endgroup$
    – Allawonder
    Apr 19 at 8:58










  • $begingroup$
    Related (maybe): math.stackexchange.com/questions/2491184/…
    $endgroup$
    – Hans Lundmark
    Apr 19 at 11:09










  • $begingroup$
    If you think it is just an approximation, can you give some bounds on the error this approximation has?
    $endgroup$
    – Hagen von Eitzen
    Apr 19 at 11:25










  • $begingroup$
    Possible duplicate (also unanswered): math.stackexchange.com/q/1053482/243259
    $endgroup$
    – Quelklef
    Apr 19 at 13:07


















  • $begingroup$
    "But in limits we plug $x=a$ ..." --- This is only a technique that can sometimes be applied. See this answer.
    $endgroup$
    – Dave L. Renfro
    Apr 19 at 7:46












  • $begingroup$
    First off, we don't plug anything. That's just one heuristic technique for evaluating limits, and it works only for continuous functions. As to your question, limiting procedures give a useful way for extending concepts in a natural way (in a way that completes a class of objects) and thus help us naturally assign values to objects that don't have values in the usual sense of finite operations. In courses in analysis, it is shown that these limiting values behave essentially like their finite-procedure counterpart, and that they generalise their domain of application. These are both desirable.
    $endgroup$
    – Allawonder
    Apr 19 at 8:58










  • $begingroup$
    Related (maybe): math.stackexchange.com/questions/2491184/…
    $endgroup$
    – Hans Lundmark
    Apr 19 at 11:09










  • $begingroup$
    If you think it is just an approximation, can you give some bounds on the error this approximation has?
    $endgroup$
    – Hagen von Eitzen
    Apr 19 at 11:25










  • $begingroup$
    Possible duplicate (also unanswered): math.stackexchange.com/q/1053482/243259
    $endgroup$
    – Quelklef
    Apr 19 at 13:07
















$begingroup$
"But in limits we plug $x=a$ ..." --- This is only a technique that can sometimes be applied. See this answer.
$endgroup$
– Dave L. Renfro
Apr 19 at 7:46






$begingroup$
"But in limits we plug $x=a$ ..." --- This is only a technique that can sometimes be applied. See this answer.
$endgroup$
– Dave L. Renfro
Apr 19 at 7:46














$begingroup$
First off, we don't plug anything. That's just one heuristic technique for evaluating limits, and it works only for continuous functions. As to your question, limiting procedures give a useful way for extending concepts in a natural way (in a way that completes a class of objects) and thus help us naturally assign values to objects that don't have values in the usual sense of finite operations. In courses in analysis, it is shown that these limiting values behave essentially like their finite-procedure counterpart, and that they generalise their domain of application. These are both desirable.
$endgroup$
– Allawonder
Apr 19 at 8:58




$begingroup$
First off, we don't plug anything. That's just one heuristic technique for evaluating limits, and it works only for continuous functions. As to your question, limiting procedures give a useful way for extending concepts in a natural way (in a way that completes a class of objects) and thus help us naturally assign values to objects that don't have values in the usual sense of finite operations. In courses in analysis, it is shown that these limiting values behave essentially like their finite-procedure counterpart, and that they generalise their domain of application. These are both desirable.
$endgroup$
– Allawonder
Apr 19 at 8:58












$begingroup$
Related (maybe): math.stackexchange.com/questions/2491184/…
$endgroup$
– Hans Lundmark
Apr 19 at 11:09




$begingroup$
Related (maybe): math.stackexchange.com/questions/2491184/…
$endgroup$
– Hans Lundmark
Apr 19 at 11:09












$begingroup$
If you think it is just an approximation, can you give some bounds on the error this approximation has?
$endgroup$
– Hagen von Eitzen
Apr 19 at 11:25




$begingroup$
If you think it is just an approximation, can you give some bounds on the error this approximation has?
$endgroup$
– Hagen von Eitzen
Apr 19 at 11:25












$begingroup$
Possible duplicate (also unanswered): math.stackexchange.com/q/1053482/243259
$endgroup$
– Quelklef
Apr 19 at 13:07




$begingroup$
Possible duplicate (also unanswered): math.stackexchange.com/q/1053482/243259
$endgroup$
– Quelklef
Apr 19 at 13:07










6 Answers
6






active

oldest

votes


















3












$begingroup$

I think I understand where you're coming from. I had the same reaction! Tangent lines should exist regardless of limits and derivatives. So, when we construct the tangent line via the derivative, we should be able to check that it's correct.



I think we're both wrong—tangent lines don't rigorously exist independent of derivatives. There are too many edge cases and difficulties when you try to define them, so we either end up with 1) no definition, which gives us no way to rigorously check that the derivative is correct; or 2) we use the derivative as our definition.



To see the difficulties involved, consider $f(x)=x^3$. We'll call the tangent line $ell$, tangent at $x_0$.



We want, of course, for $ell$ to touch $f$ at $x_0$. But we want it to do so ONLY once within a small neighborhood. So perhaps we could use this as the definition of a tangent line.



However, secant lines also fit this definition! So it can't be correct. To fix this, perhaps we note that on the right side of $f$, the tangent line is always below the curve in a small neighborhood, and on the left side, it's above, and at $x_0=0$, the tangent line is just flat. Perhaps we could restrict the tangent line to abide by this—below on right, above on left, flat in middle—and have that amendment make our definition.



And that would work, I suppose. But it's an ugly piecewise definition, and, more importantly, it doesn't generalize well. To tell where the tangent line should be 'above' or 'below' uses the concept of concavity, which we need derivatives for. So we are able to rigorously define a tangent line without derivatives for $x^3$, but not in general.



The tangent line is a deceptively difficult concept to define. There isn't (as far as I know) a generally accepted definition besides derivatives that we can use to verify that derivatives give the correct results. You can always check for each individual problem you come across by eyeballing what the tangent line should be, but doing it in general is hard.






share|cite|improve this answer











$endgroup$





















    3












    $begingroup$

    A line-by-line correction of OP that should answer the confusions therein:



    (1) Limits tell us how a function behaves nearby $x=a.$ The function doesn't even have to be defined at $a.$ If it is also defined there, and it coincides with the limit, then the limit in addition happens to tell us what's happening at $a,$ contrary to your first sentence. But the important thing to note is that limits tell us how a function is behaving in the neighborhood of some point in its domain.



    (2) From above, it follows that $f(a)$ need not exist at all in order for $lim_{xto a}{f(x)}$ to exist. Indeed, even if $f(a)$ exists, it need not coincide with the limiting value. In essence, limiting values have nothing essentially to do with values. But in cases where $lim_{xto a}{f(x)}$ exists and coincides with $f(a),$ we may use this fact to evaluate the limiting value, but it is no way an approximation. There's nothing about approximations here. The conditions are all equalities. Again, the salient point here is that we can't always evaluate limits, when they exist, by a substitution as you think. This only applies to functions continuous at the point in question.



    (3) You have yet to understand the concept of limit properly. It is not an approximation to anything, but rather that towards which we approximate; thus by definition it will sometimes transcend every approximant. Rather, limits are a way to extend usual concepts into wider domains of applicability in such a way that the new objects properly generalise the old -- that is, they possess all important properties of the old objects, and contain the latter as a subclass. In particular here, bringing in limiting operations helps us to assign a definite meaning to some sequences of approximations that distinguish themselves from others. Note that whereas some terms of these types of sequence approximate the limiting value, this value itself is decidedly not an approximation, but a uniquely defined value, fixed by this sequence (and an assumption about bounded, nonempty subsets of real numbers that genuinely and smoothly extends the concept of maximum or minimum values of bounded, nonempty subsets of numbers). This well-defined limiting value, whenever it exists, has been shown many times over to legitimately (in a sense that can be made precise) extend the domain of application of the concept under study. Here, it follows that whenever a sequence of approximating slopes converges, we can choose to call (by a justified extension) the limit a slope too, indeed the slope, defined by that sequence. That's simply what it means to work with real numbers. If you've not understood this property (technically called completeness) then you've not begun to understand the real numbers.






    share|cite|improve this answer











    $endgroup$













    • $begingroup$
      I think the "tangent line" in the title is a hint that the question should have been answered at a simpler level. Yes, there's no need for f(a) to exist, or to equal the limit. But students which are still working with the notion of tangent lines are unlikely to be dealing with discontinuities, I think.
      $endgroup$
      – MSalters
      Apr 19 at 15:41










    • $begingroup$
      "If it is also defined there, then the limit tells us in addition what's happening at $a$" - The limit itself never tells us what is happening at $a$. If $f$ is continuous at $a$, then the limit is indeed what is happening at $a$, but it was the condition of continuity that told us this, not the limit.
      $endgroup$
      – Paul Sinclair
      Apr 19 at 16:43










    • $begingroup$
      @PaulSinclair I would agree that the limit never tells what's happening at $a.$ As I emphasized, the main point is that the limit tells what's happening close to $a.$ However, I think it was something like what's in your second paragraph I was trying to express. But I think the emphases I made should point the way. In any case I've edited the relevant part. Thanks.
      $endgroup$
      – Allawonder
      Apr 19 at 18:54










    • $begingroup$
      @MSalters I don't understand what you mean by simpler level. My emphases couldn't have been simpler. What I noticed was the erroneous notion of limiting procedures in OP, which I think was the source of the confusions, and this I attempted to set right. Once OP understands that whenever a sequence converges, then its limit is unique, and they also see that limits are a way to complete $mathbf Q,$ and they also agree to call the limit of a sequence of secant slopes a tangent slope, then everything falls into place.
      $endgroup$
      – Allawonder
      Apr 19 at 19:00



















    2












    $begingroup$

    I think the problem with your thinking is that, you are assuming that there is something that is predefined by "god" which is the slope at a point that has exact value, and we are trying to reach it by the processes of taking a limit, and we can never reach that value. But that is not the case. What we do, is that we define the notion of the slope at a point , by the processes of taking a limit. And what ever the exact value that comes out of the limit, is what we call the slope at that point.



    I guess that what is bothering you, is the notion of a limit that you learn at first-year calculus , and i felt the same when i first learn it, because at some stage of the problem we say that x approaches some value a , then at another stage we substitute a for x. But to understand what is actually going on, you have to understand the epsilon-delta definition of a limit.
    see:https://en.wikipedia.org/wiki/(%CE%B5,_%CE%B4)-definition_of_limit






    share|cite|improve this answer











    $endgroup$





















      1












      $begingroup$

      The idea is that the approximation gets better and better as $x$ gets closer to $a$, and, so to say, at $x=a$ it is no more approximative.



      Below, a function in blue, the tangent in green and the difference in magenta. As you see, the difference decreases very quickly close to the tangency point.



      enter image description here



      This is not achieved with a different slope.



      enter image description here






      share|cite|improve this answer











      $endgroup$





















        1












        $begingroup$

        Thought I'd add something with an emphasis on geometry to the excellent answers above.



        Consider a point A on a circle and a point B elsewhere on the circle and draw a line between the two. The line touches the circle in two places. We have a secant line. Select another point B closer to A than the original B and draw another line. Again it intersects in two places, another secant line. Iterate bringing the new Points be ever closer to we get more and more secant lines until B is actually on A and the resulting line 1) intersects the circle at one point, and 2) is perpendicular to the radius through A. We have a tangent to our circle. We can associate a slope to the corresponding line.



        To many curves we can associate a tangent circle, any circle that pass through the curve at only one point. Then the slope of the tangent line to the circle is the slope of the tangent line to the curve. We have the slope of a tangent line without calculus.



        Consider the parabola $y=x^2$. Let's create a circle that intersects it at only one point.



        $(x-a)^2+(x^2-b)^2=r^2$



        $x^2+a^2-2ax+x^4+b^2-2bx^2=r^2$



        $x^4+(1-2b)x^2-2ax+(a^2+b^2-r^2)=0$



        Under what conditions does this have only one solution on the parabola?



        We can make r arbitrarily small and find an (a,b) within r of the target point, $x_0$. So we can make the constant term of the polynomial vanish reducing it to



        $x^3+(1-2b)x-2a=0$



        We want the pont who's tangency we want to find to be a solution so we divide by $(x-x_0)$.



        We want $x_0$ to have multiplicity 3 or the quadratic to have no real solutions. This tells us the relationship between the center of our circle, the radius and the point of tangency.






        share|cite|improve this answer











        $endgroup$





















          0












          $begingroup$

          Well, if you have two points $(x_1,y_1)$ and $(x_2,y_2)$ in the plane, then the slope of the unique line through these points is
          $$ m = frac{y_2-y_1}{x_2-x_1}.$$
          If you consider a (differentiable) function $f:{Bbb R}rightarrow{Bbb R}$, then the slope of the line between $(x_1,f(x_1))$ and $(x_2,f(x_2))$ is
          $$ m = frac{f(x_2)-f(x_1)}{x_2-x_1}.$$
          If you take the points $x_0$ and $x$ at the x-axis, then consider the quotient
          $$frac{f(x)-f(x_0)}{x-x_0}$$
          when $x$ moves to $x_0$: $xrightarrow x_0$.
          Then this quotient becomes (in the limit) the slope of the tangent line of $f$ at the point $x_0$. This is particularly easy to understand if you draw a picture.



          Denote the slope in the limit as $f'(x_0)$. Then the above quotient gives
          $$f'(x_0) = frac{f(x)-f(x_0)}{x-x_0}$$
          and so the equation of the tangent line
          $$f(x) = f'(x_0)(x-x_0) + f(x_0).$$






          share|cite|improve this answer











          $endgroup$













          • $begingroup$
            Since $x_0$ is an approximate , Then why we get the exact value of the slope ?
            $endgroup$
            – Mohammad Alshareef
            Apr 19 at 7:25








          • 1




            $begingroup$
            @Mohammad Alshareef: The "exact value of the slope at a given point" is defined using a limit, or do you have another way of defining it? Your question seems to me like asking why we use three letters to write the word "big". (Answer: Because the word "big" is defined by making use of three letters, or something to this effect.)
            $endgroup$
            – Dave L. Renfro
            Apr 19 at 7:42










          • $begingroup$
            @DaveL.Renfro : Does $f(xto a) = f(x=a)$ ?
            $endgroup$
            – Mohammad Alshareef
            Apr 19 at 8:06












          • $begingroup$
            Yes, but that's because (i) the function $g(x) = x$ is continuous [note that your two inputs are "$x rightarrow a$" and "$a$"; these are equal because of (i)] and (ii) $f$ is (presumably) a function [$f$ a function implies that whenever $c=d,$ then we have $f(c) = f(d)].$ Perhaps you wanted to say "limit as $x rightarrow a$ of $f(x)$" is equal to "$f(a)$"? (This is not true, however, unless $f$ is continuous at $x=0.)$
            $endgroup$
            – Dave L. Renfro
            Apr 19 at 8:13












          • $begingroup$
            Actually ,This explain everything ,But I am not satisfied , Since $(xto a) + h = a$ , $h ≠ 0$ , Then $f(xto a)$ should not be equal to $f(a)$.
            $endgroup$
            – Mohammad Alshareef
            Apr 19 at 8:24














          Your Answer








          StackExchange.ready(function() {
          var channelOptions = {
          tags: "".split(" "),
          id: "69"
          };
          initTagRenderer("".split(" "), "".split(" "), channelOptions);

          StackExchange.using("externalEditor", function() {
          // Have to fire editor after snippets, if snippets enabled
          if (StackExchange.settings.snippets.snippetsEnabled) {
          StackExchange.using("snippets", function() {
          createEditor();
          });
          }
          else {
          createEditor();
          }
          });

          function createEditor() {
          StackExchange.prepareEditor({
          heartbeatType: 'answer',
          autoActivateHeartbeat: false,
          convertImagesToLinks: true,
          noModals: true,
          showLowRepImageUploadWarning: true,
          reputationToPostImages: 10,
          bindNavPrevention: true,
          postfix: "",
          imageUploader: {
          brandingHtml: "Powered by u003ca class="icon-imgur-white" href="https://imgur.com/"u003eu003c/au003e",
          contentPolicyHtml: "User contributions licensed under u003ca href="https://creativecommons.org/licenses/by-sa/3.0/"u003ecc by-sa 3.0 with attribution requiredu003c/au003e u003ca href="https://stackoverflow.com/legal/content-policy"u003e(content policy)u003c/au003e",
          allowUrls: true
          },
          noCode: true, onDemand: true,
          discardSelector: ".discard-answer"
          ,immediatelyShowMarkdownHelp:true
          });


          }
          });














          draft saved

          draft discarded


















          StackExchange.ready(
          function () {
          StackExchange.openid.initPostLogin('.new-post-login', 'https%3a%2f%2fmath.stackexchange.com%2fquestions%2f3193260%2fwhy-limits-give-us-the-exact-value-of-the-slope-of-the-tangent-line%23new-answer', 'question_page');
          }
          );

          Post as a guest















          Required, but never shown

























          6 Answers
          6






          active

          oldest

          votes








          6 Answers
          6






          active

          oldest

          votes









          active

          oldest

          votes






          active

          oldest

          votes









          3












          $begingroup$

          I think I understand where you're coming from. I had the same reaction! Tangent lines should exist regardless of limits and derivatives. So, when we construct the tangent line via the derivative, we should be able to check that it's correct.



          I think we're both wrong—tangent lines don't rigorously exist independent of derivatives. There are too many edge cases and difficulties when you try to define them, so we either end up with 1) no definition, which gives us no way to rigorously check that the derivative is correct; or 2) we use the derivative as our definition.



          To see the difficulties involved, consider $f(x)=x^3$. We'll call the tangent line $ell$, tangent at $x_0$.



          We want, of course, for $ell$ to touch $f$ at $x_0$. But we want it to do so ONLY once within a small neighborhood. So perhaps we could use this as the definition of a tangent line.



          However, secant lines also fit this definition! So it can't be correct. To fix this, perhaps we note that on the right side of $f$, the tangent line is always below the curve in a small neighborhood, and on the left side, it's above, and at $x_0=0$, the tangent line is just flat. Perhaps we could restrict the tangent line to abide by this—below on right, above on left, flat in middle—and have that amendment make our definition.



          And that would work, I suppose. But it's an ugly piecewise definition, and, more importantly, it doesn't generalize well. To tell where the tangent line should be 'above' or 'below' uses the concept of concavity, which we need derivatives for. So we are able to rigorously define a tangent line without derivatives for $x^3$, but not in general.



          The tangent line is a deceptively difficult concept to define. There isn't (as far as I know) a generally accepted definition besides derivatives that we can use to verify that derivatives give the correct results. You can always check for each individual problem you come across by eyeballing what the tangent line should be, but doing it in general is hard.






          share|cite|improve this answer











          $endgroup$


















            3












            $begingroup$

            I think I understand where you're coming from. I had the same reaction! Tangent lines should exist regardless of limits and derivatives. So, when we construct the tangent line via the derivative, we should be able to check that it's correct.



            I think we're both wrong—tangent lines don't rigorously exist independent of derivatives. There are too many edge cases and difficulties when you try to define them, so we either end up with 1) no definition, which gives us no way to rigorously check that the derivative is correct; or 2) we use the derivative as our definition.



            To see the difficulties involved, consider $f(x)=x^3$. We'll call the tangent line $ell$, tangent at $x_0$.



            We want, of course, for $ell$ to touch $f$ at $x_0$. But we want it to do so ONLY once within a small neighborhood. So perhaps we could use this as the definition of a tangent line.



            However, secant lines also fit this definition! So it can't be correct. To fix this, perhaps we note that on the right side of $f$, the tangent line is always below the curve in a small neighborhood, and on the left side, it's above, and at $x_0=0$, the tangent line is just flat. Perhaps we could restrict the tangent line to abide by this—below on right, above on left, flat in middle—and have that amendment make our definition.



            And that would work, I suppose. But it's an ugly piecewise definition, and, more importantly, it doesn't generalize well. To tell where the tangent line should be 'above' or 'below' uses the concept of concavity, which we need derivatives for. So we are able to rigorously define a tangent line without derivatives for $x^3$, but not in general.



            The tangent line is a deceptively difficult concept to define. There isn't (as far as I know) a generally accepted definition besides derivatives that we can use to verify that derivatives give the correct results. You can always check for each individual problem you come across by eyeballing what the tangent line should be, but doing it in general is hard.






            share|cite|improve this answer











            $endgroup$
















              3












              3








              3





              $begingroup$

              I think I understand where you're coming from. I had the same reaction! Tangent lines should exist regardless of limits and derivatives. So, when we construct the tangent line via the derivative, we should be able to check that it's correct.



              I think we're both wrong—tangent lines don't rigorously exist independent of derivatives. There are too many edge cases and difficulties when you try to define them, so we either end up with 1) no definition, which gives us no way to rigorously check that the derivative is correct; or 2) we use the derivative as our definition.



              To see the difficulties involved, consider $f(x)=x^3$. We'll call the tangent line $ell$, tangent at $x_0$.



              We want, of course, for $ell$ to touch $f$ at $x_0$. But we want it to do so ONLY once within a small neighborhood. So perhaps we could use this as the definition of a tangent line.



              However, secant lines also fit this definition! So it can't be correct. To fix this, perhaps we note that on the right side of $f$, the tangent line is always below the curve in a small neighborhood, and on the left side, it's above, and at $x_0=0$, the tangent line is just flat. Perhaps we could restrict the tangent line to abide by this—below on right, above on left, flat in middle—and have that amendment make our definition.



              And that would work, I suppose. But it's an ugly piecewise definition, and, more importantly, it doesn't generalize well. To tell where the tangent line should be 'above' or 'below' uses the concept of concavity, which we need derivatives for. So we are able to rigorously define a tangent line without derivatives for $x^3$, but not in general.



              The tangent line is a deceptively difficult concept to define. There isn't (as far as I know) a generally accepted definition besides derivatives that we can use to verify that derivatives give the correct results. You can always check for each individual problem you come across by eyeballing what the tangent line should be, but doing it in general is hard.






              share|cite|improve this answer











              $endgroup$



              I think I understand where you're coming from. I had the same reaction! Tangent lines should exist regardless of limits and derivatives. So, when we construct the tangent line via the derivative, we should be able to check that it's correct.



              I think we're both wrong—tangent lines don't rigorously exist independent of derivatives. There are too many edge cases and difficulties when you try to define them, so we either end up with 1) no definition, which gives us no way to rigorously check that the derivative is correct; or 2) we use the derivative as our definition.



              To see the difficulties involved, consider $f(x)=x^3$. We'll call the tangent line $ell$, tangent at $x_0$.



              We want, of course, for $ell$ to touch $f$ at $x_0$. But we want it to do so ONLY once within a small neighborhood. So perhaps we could use this as the definition of a tangent line.



              However, secant lines also fit this definition! So it can't be correct. To fix this, perhaps we note that on the right side of $f$, the tangent line is always below the curve in a small neighborhood, and on the left side, it's above, and at $x_0=0$, the tangent line is just flat. Perhaps we could restrict the tangent line to abide by this—below on right, above on left, flat in middle—and have that amendment make our definition.



              And that would work, I suppose. But it's an ugly piecewise definition, and, more importantly, it doesn't generalize well. To tell where the tangent line should be 'above' or 'below' uses the concept of concavity, which we need derivatives for. So we are able to rigorously define a tangent line without derivatives for $x^3$, but not in general.



              The tangent line is a deceptively difficult concept to define. There isn't (as far as I know) a generally accepted definition besides derivatives that we can use to verify that derivatives give the correct results. You can always check for each individual problem you come across by eyeballing what the tangent line should be, but doing it in general is hard.







              share|cite|improve this answer














              share|cite|improve this answer



              share|cite|improve this answer








              edited Apr 19 at 13:39

























              answered Apr 19 at 13:32









              QuelklefQuelklef

              465311




              465311























                  3












                  $begingroup$

                  A line-by-line correction of OP that should answer the confusions therein:



                  (1) Limits tell us how a function behaves nearby $x=a.$ The function doesn't even have to be defined at $a.$ If it is also defined there, and it coincides with the limit, then the limit in addition happens to tell us what's happening at $a,$ contrary to your first sentence. But the important thing to note is that limits tell us how a function is behaving in the neighborhood of some point in its domain.



                  (2) From above, it follows that $f(a)$ need not exist at all in order for $lim_{xto a}{f(x)}$ to exist. Indeed, even if $f(a)$ exists, it need not coincide with the limiting value. In essence, limiting values have nothing essentially to do with values. But in cases where $lim_{xto a}{f(x)}$ exists and coincides with $f(a),$ we may use this fact to evaluate the limiting value, but it is no way an approximation. There's nothing about approximations here. The conditions are all equalities. Again, the salient point here is that we can't always evaluate limits, when they exist, by a substitution as you think. This only applies to functions continuous at the point in question.



                  (3) You have yet to understand the concept of limit properly. It is not an approximation to anything, but rather that towards which we approximate; thus by definition it will sometimes transcend every approximant. Rather, limits are a way to extend usual concepts into wider domains of applicability in such a way that the new objects properly generalise the old -- that is, they possess all important properties of the old objects, and contain the latter as a subclass. In particular here, bringing in limiting operations helps us to assign a definite meaning to some sequences of approximations that distinguish themselves from others. Note that whereas some terms of these types of sequence approximate the limiting value, this value itself is decidedly not an approximation, but a uniquely defined value, fixed by this sequence (and an assumption about bounded, nonempty subsets of real numbers that genuinely and smoothly extends the concept of maximum or minimum values of bounded, nonempty subsets of numbers). This well-defined limiting value, whenever it exists, has been shown many times over to legitimately (in a sense that can be made precise) extend the domain of application of the concept under study. Here, it follows that whenever a sequence of approximating slopes converges, we can choose to call (by a justified extension) the limit a slope too, indeed the slope, defined by that sequence. That's simply what it means to work with real numbers. If you've not understood this property (technically called completeness) then you've not begun to understand the real numbers.






                  share|cite|improve this answer











                  $endgroup$













                  • $begingroup$
                    I think the "tangent line" in the title is a hint that the question should have been answered at a simpler level. Yes, there's no need for f(a) to exist, or to equal the limit. But students which are still working with the notion of tangent lines are unlikely to be dealing with discontinuities, I think.
                    $endgroup$
                    – MSalters
                    Apr 19 at 15:41










                  • $begingroup$
                    "If it is also defined there, then the limit tells us in addition what's happening at $a$" - The limit itself never tells us what is happening at $a$. If $f$ is continuous at $a$, then the limit is indeed what is happening at $a$, but it was the condition of continuity that told us this, not the limit.
                    $endgroup$
                    – Paul Sinclair
                    Apr 19 at 16:43










                  • $begingroup$
                    @PaulSinclair I would agree that the limit never tells what's happening at $a.$ As I emphasized, the main point is that the limit tells what's happening close to $a.$ However, I think it was something like what's in your second paragraph I was trying to express. But I think the emphases I made should point the way. In any case I've edited the relevant part. Thanks.
                    $endgroup$
                    – Allawonder
                    Apr 19 at 18:54










                  • $begingroup$
                    @MSalters I don't understand what you mean by simpler level. My emphases couldn't have been simpler. What I noticed was the erroneous notion of limiting procedures in OP, which I think was the source of the confusions, and this I attempted to set right. Once OP understands that whenever a sequence converges, then its limit is unique, and they also see that limits are a way to complete $mathbf Q,$ and they also agree to call the limit of a sequence of secant slopes a tangent slope, then everything falls into place.
                    $endgroup$
                    – Allawonder
                    Apr 19 at 19:00
















                  3












                  $begingroup$

                  A line-by-line correction of OP that should answer the confusions therein:



                  (1) Limits tell us how a function behaves nearby $x=a.$ The function doesn't even have to be defined at $a.$ If it is also defined there, and it coincides with the limit, then the limit in addition happens to tell us what's happening at $a,$ contrary to your first sentence. But the important thing to note is that limits tell us how a function is behaving in the neighborhood of some point in its domain.



                  (2) From above, it follows that $f(a)$ need not exist at all in order for $lim_{xto a}{f(x)}$ to exist. Indeed, even if $f(a)$ exists, it need not coincide with the limiting value. In essence, limiting values have nothing essentially to do with values. But in cases where $lim_{xto a}{f(x)}$ exists and coincides with $f(a),$ we may use this fact to evaluate the limiting value, but it is no way an approximation. There's nothing about approximations here. The conditions are all equalities. Again, the salient point here is that we can't always evaluate limits, when they exist, by a substitution as you think. This only applies to functions continuous at the point in question.



                  (3) You have yet to understand the concept of limit properly. It is not an approximation to anything, but rather that towards which we approximate; thus by definition it will sometimes transcend every approximant. Rather, limits are a way to extend usual concepts into wider domains of applicability in such a way that the new objects properly generalise the old -- that is, they possess all important properties of the old objects, and contain the latter as a subclass. In particular here, bringing in limiting operations helps us to assign a definite meaning to some sequences of approximations that distinguish themselves from others. Note that whereas some terms of these types of sequence approximate the limiting value, this value itself is decidedly not an approximation, but a uniquely defined value, fixed by this sequence (and an assumption about bounded, nonempty subsets of real numbers that genuinely and smoothly extends the concept of maximum or minimum values of bounded, nonempty subsets of numbers). This well-defined limiting value, whenever it exists, has been shown many times over to legitimately (in a sense that can be made precise) extend the domain of application of the concept under study. Here, it follows that whenever a sequence of approximating slopes converges, we can choose to call (by a justified extension) the limit a slope too, indeed the slope, defined by that sequence. That's simply what it means to work with real numbers. If you've not understood this property (technically called completeness) then you've not begun to understand the real numbers.






                  share|cite|improve this answer











                  $endgroup$













                  • $begingroup$
                    I think the "tangent line" in the title is a hint that the question should have been answered at a simpler level. Yes, there's no need for f(a) to exist, or to equal the limit. But students which are still working with the notion of tangent lines are unlikely to be dealing with discontinuities, I think.
                    $endgroup$
                    – MSalters
                    Apr 19 at 15:41










                  • $begingroup$
                    "If it is also defined there, then the limit tells us in addition what's happening at $a$" - The limit itself never tells us what is happening at $a$. If $f$ is continuous at $a$, then the limit is indeed what is happening at $a$, but it was the condition of continuity that told us this, not the limit.
                    $endgroup$
                    – Paul Sinclair
                    Apr 19 at 16:43










                  • $begingroup$
                    @PaulSinclair I would agree that the limit never tells what's happening at $a.$ As I emphasized, the main point is that the limit tells what's happening close to $a.$ However, I think it was something like what's in your second paragraph I was trying to express. But I think the emphases I made should point the way. In any case I've edited the relevant part. Thanks.
                    $endgroup$
                    – Allawonder
                    Apr 19 at 18:54










                  • $begingroup$
                    @MSalters I don't understand what you mean by simpler level. My emphases couldn't have been simpler. What I noticed was the erroneous notion of limiting procedures in OP, which I think was the source of the confusions, and this I attempted to set right. Once OP understands that whenever a sequence converges, then its limit is unique, and they also see that limits are a way to complete $mathbf Q,$ and they also agree to call the limit of a sequence of secant slopes a tangent slope, then everything falls into place.
                    $endgroup$
                    – Allawonder
                    Apr 19 at 19:00














                  3












                  3








                  3





                  $begingroup$

                  A line-by-line correction of OP that should answer the confusions therein:



                  (1) Limits tell us how a function behaves nearby $x=a.$ The function doesn't even have to be defined at $a.$ If it is also defined there, and it coincides with the limit, then the limit in addition happens to tell us what's happening at $a,$ contrary to your first sentence. But the important thing to note is that limits tell us how a function is behaving in the neighborhood of some point in its domain.



                  (2) From above, it follows that $f(a)$ need not exist at all in order for $lim_{xto a}{f(x)}$ to exist. Indeed, even if $f(a)$ exists, it need not coincide with the limiting value. In essence, limiting values have nothing essentially to do with values. But in cases where $lim_{xto a}{f(x)}$ exists and coincides with $f(a),$ we may use this fact to evaluate the limiting value, but it is no way an approximation. There's nothing about approximations here. The conditions are all equalities. Again, the salient point here is that we can't always evaluate limits, when they exist, by a substitution as you think. This only applies to functions continuous at the point in question.



                  (3) You have yet to understand the concept of limit properly. It is not an approximation to anything, but rather that towards which we approximate; thus by definition it will sometimes transcend every approximant. Rather, limits are a way to extend usual concepts into wider domains of applicability in such a way that the new objects properly generalise the old -- that is, they possess all important properties of the old objects, and contain the latter as a subclass. In particular here, bringing in limiting operations helps us to assign a definite meaning to some sequences of approximations that distinguish themselves from others. Note that whereas some terms of these types of sequence approximate the limiting value, this value itself is decidedly not an approximation, but a uniquely defined value, fixed by this sequence (and an assumption about bounded, nonempty subsets of real numbers that genuinely and smoothly extends the concept of maximum or minimum values of bounded, nonempty subsets of numbers). This well-defined limiting value, whenever it exists, has been shown many times over to legitimately (in a sense that can be made precise) extend the domain of application of the concept under study. Here, it follows that whenever a sequence of approximating slopes converges, we can choose to call (by a justified extension) the limit a slope too, indeed the slope, defined by that sequence. That's simply what it means to work with real numbers. If you've not understood this property (technically called completeness) then you've not begun to understand the real numbers.






                  share|cite|improve this answer











                  $endgroup$



                  A line-by-line correction of OP that should answer the confusions therein:



                  (1) Limits tell us how a function behaves nearby $x=a.$ The function doesn't even have to be defined at $a.$ If it is also defined there, and it coincides with the limit, then the limit in addition happens to tell us what's happening at $a,$ contrary to your first sentence. But the important thing to note is that limits tell us how a function is behaving in the neighborhood of some point in its domain.



                  (2) From above, it follows that $f(a)$ need not exist at all in order for $lim_{xto a}{f(x)}$ to exist. Indeed, even if $f(a)$ exists, it need not coincide with the limiting value. In essence, limiting values have nothing essentially to do with values. But in cases where $lim_{xto a}{f(x)}$ exists and coincides with $f(a),$ we may use this fact to evaluate the limiting value, but it is no way an approximation. There's nothing about approximations here. The conditions are all equalities. Again, the salient point here is that we can't always evaluate limits, when they exist, by a substitution as you think. This only applies to functions continuous at the point in question.



                  (3) You have yet to understand the concept of limit properly. It is not an approximation to anything, but rather that towards which we approximate; thus by definition it will sometimes transcend every approximant. Rather, limits are a way to extend usual concepts into wider domains of applicability in such a way that the new objects properly generalise the old -- that is, they possess all important properties of the old objects, and contain the latter as a subclass. In particular here, bringing in limiting operations helps us to assign a definite meaning to some sequences of approximations that distinguish themselves from others. Note that whereas some terms of these types of sequence approximate the limiting value, this value itself is decidedly not an approximation, but a uniquely defined value, fixed by this sequence (and an assumption about bounded, nonempty subsets of real numbers that genuinely and smoothly extends the concept of maximum or minimum values of bounded, nonempty subsets of numbers). This well-defined limiting value, whenever it exists, has been shown many times over to legitimately (in a sense that can be made precise) extend the domain of application of the concept under study. Here, it follows that whenever a sequence of approximating slopes converges, we can choose to call (by a justified extension) the limit a slope too, indeed the slope, defined by that sequence. That's simply what it means to work with real numbers. If you've not understood this property (technically called completeness) then you've not begun to understand the real numbers.







                  share|cite|improve this answer














                  share|cite|improve this answer



                  share|cite|improve this answer








                  edited Apr 19 at 18:54

























                  answered Apr 19 at 9:27









                  AllawonderAllawonder

                  2,364717




                  2,364717












                  • $begingroup$
                    I think the "tangent line" in the title is a hint that the question should have been answered at a simpler level. Yes, there's no need for f(a) to exist, or to equal the limit. But students which are still working with the notion of tangent lines are unlikely to be dealing with discontinuities, I think.
                    $endgroup$
                    – MSalters
                    Apr 19 at 15:41










                  • $begingroup$
                    "If it is also defined there, then the limit tells us in addition what's happening at $a$" - The limit itself never tells us what is happening at $a$. If $f$ is continuous at $a$, then the limit is indeed what is happening at $a$, but it was the condition of continuity that told us this, not the limit.
                    $endgroup$
                    – Paul Sinclair
                    Apr 19 at 16:43










                  • $begingroup$
                    @PaulSinclair I would agree that the limit never tells what's happening at $a.$ As I emphasized, the main point is that the limit tells what's happening close to $a.$ However, I think it was something like what's in your second paragraph I was trying to express. But I think the emphases I made should point the way. In any case I've edited the relevant part. Thanks.
                    $endgroup$
                    – Allawonder
                    Apr 19 at 18:54










                  • $begingroup$
                    @MSalters I don't understand what you mean by simpler level. My emphases couldn't have been simpler. What I noticed was the erroneous notion of limiting procedures in OP, which I think was the source of the confusions, and this I attempted to set right. Once OP understands that whenever a sequence converges, then its limit is unique, and they also see that limits are a way to complete $mathbf Q,$ and they also agree to call the limit of a sequence of secant slopes a tangent slope, then everything falls into place.
                    $endgroup$
                    – Allawonder
                    Apr 19 at 19:00


















                  • $begingroup$
                    I think the "tangent line" in the title is a hint that the question should have been answered at a simpler level. Yes, there's no need for f(a) to exist, or to equal the limit. But students which are still working with the notion of tangent lines are unlikely to be dealing with discontinuities, I think.
                    $endgroup$
                    – MSalters
                    Apr 19 at 15:41










                  • $begingroup$
                    "If it is also defined there, then the limit tells us in addition what's happening at $a$" - The limit itself never tells us what is happening at $a$. If $f$ is continuous at $a$, then the limit is indeed what is happening at $a$, but it was the condition of continuity that told us this, not the limit.
                    $endgroup$
                    – Paul Sinclair
                    Apr 19 at 16:43










                  • $begingroup$
                    @PaulSinclair I would agree that the limit never tells what's happening at $a.$ As I emphasized, the main point is that the limit tells what's happening close to $a.$ However, I think it was something like what's in your second paragraph I was trying to express. But I think the emphases I made should point the way. In any case I've edited the relevant part. Thanks.
                    $endgroup$
                    – Allawonder
                    Apr 19 at 18:54










                  • $begingroup$
                    @MSalters I don't understand what you mean by simpler level. My emphases couldn't have been simpler. What I noticed was the erroneous notion of limiting procedures in OP, which I think was the source of the confusions, and this I attempted to set right. Once OP understands that whenever a sequence converges, then its limit is unique, and they also see that limits are a way to complete $mathbf Q,$ and they also agree to call the limit of a sequence of secant slopes a tangent slope, then everything falls into place.
                    $endgroup$
                    – Allawonder
                    Apr 19 at 19:00
















                  $begingroup$
                  I think the "tangent line" in the title is a hint that the question should have been answered at a simpler level. Yes, there's no need for f(a) to exist, or to equal the limit. But students which are still working with the notion of tangent lines are unlikely to be dealing with discontinuities, I think.
                  $endgroup$
                  – MSalters
                  Apr 19 at 15:41




                  $begingroup$
                  I think the "tangent line" in the title is a hint that the question should have been answered at a simpler level. Yes, there's no need for f(a) to exist, or to equal the limit. But students which are still working with the notion of tangent lines are unlikely to be dealing with discontinuities, I think.
                  $endgroup$
                  – MSalters
                  Apr 19 at 15:41












                  $begingroup$
                  "If it is also defined there, then the limit tells us in addition what's happening at $a$" - The limit itself never tells us what is happening at $a$. If $f$ is continuous at $a$, then the limit is indeed what is happening at $a$, but it was the condition of continuity that told us this, not the limit.
                  $endgroup$
                  – Paul Sinclair
                  Apr 19 at 16:43




                  $begingroup$
                  "If it is also defined there, then the limit tells us in addition what's happening at $a$" - The limit itself never tells us what is happening at $a$. If $f$ is continuous at $a$, then the limit is indeed what is happening at $a$, but it was the condition of continuity that told us this, not the limit.
                  $endgroup$
                  – Paul Sinclair
                  Apr 19 at 16:43












                  $begingroup$
                  @PaulSinclair I would agree that the limit never tells what's happening at $a.$ As I emphasized, the main point is that the limit tells what's happening close to $a.$ However, I think it was something like what's in your second paragraph I was trying to express. But I think the emphases I made should point the way. In any case I've edited the relevant part. Thanks.
                  $endgroup$
                  – Allawonder
                  Apr 19 at 18:54




                  $begingroup$
                  @PaulSinclair I would agree that the limit never tells what's happening at $a.$ As I emphasized, the main point is that the limit tells what's happening close to $a.$ However, I think it was something like what's in your second paragraph I was trying to express. But I think the emphases I made should point the way. In any case I've edited the relevant part. Thanks.
                  $endgroup$
                  – Allawonder
                  Apr 19 at 18:54












                  $begingroup$
                  @MSalters I don't understand what you mean by simpler level. My emphases couldn't have been simpler. What I noticed was the erroneous notion of limiting procedures in OP, which I think was the source of the confusions, and this I attempted to set right. Once OP understands that whenever a sequence converges, then its limit is unique, and they also see that limits are a way to complete $mathbf Q,$ and they also agree to call the limit of a sequence of secant slopes a tangent slope, then everything falls into place.
                  $endgroup$
                  – Allawonder
                  Apr 19 at 19:00




                  $begingroup$
                  @MSalters I don't understand what you mean by simpler level. My emphases couldn't have been simpler. What I noticed was the erroneous notion of limiting procedures in OP, which I think was the source of the confusions, and this I attempted to set right. Once OP understands that whenever a sequence converges, then its limit is unique, and they also see that limits are a way to complete $mathbf Q,$ and they also agree to call the limit of a sequence of secant slopes a tangent slope, then everything falls into place.
                  $endgroup$
                  – Allawonder
                  Apr 19 at 19:00











                  2












                  $begingroup$

                  I think the problem with your thinking is that, you are assuming that there is something that is predefined by "god" which is the slope at a point that has exact value, and we are trying to reach it by the processes of taking a limit, and we can never reach that value. But that is not the case. What we do, is that we define the notion of the slope at a point , by the processes of taking a limit. And what ever the exact value that comes out of the limit, is what we call the slope at that point.



                  I guess that what is bothering you, is the notion of a limit that you learn at first-year calculus , and i felt the same when i first learn it, because at some stage of the problem we say that x approaches some value a , then at another stage we substitute a for x. But to understand what is actually going on, you have to understand the epsilon-delta definition of a limit.
                  see:https://en.wikipedia.org/wiki/(%CE%B5,_%CE%B4)-definition_of_limit






                  share|cite|improve this answer











                  $endgroup$


















                    2












                    $begingroup$

                    I think the problem with your thinking is that, you are assuming that there is something that is predefined by "god" which is the slope at a point that has exact value, and we are trying to reach it by the processes of taking a limit, and we can never reach that value. But that is not the case. What we do, is that we define the notion of the slope at a point , by the processes of taking a limit. And what ever the exact value that comes out of the limit, is what we call the slope at that point.



                    I guess that what is bothering you, is the notion of a limit that you learn at first-year calculus , and i felt the same when i first learn it, because at some stage of the problem we say that x approaches some value a , then at another stage we substitute a for x. But to understand what is actually going on, you have to understand the epsilon-delta definition of a limit.
                    see:https://en.wikipedia.org/wiki/(%CE%B5,_%CE%B4)-definition_of_limit






                    share|cite|improve this answer











                    $endgroup$
















                      2












                      2








                      2





                      $begingroup$

                      I think the problem with your thinking is that, you are assuming that there is something that is predefined by "god" which is the slope at a point that has exact value, and we are trying to reach it by the processes of taking a limit, and we can never reach that value. But that is not the case. What we do, is that we define the notion of the slope at a point , by the processes of taking a limit. And what ever the exact value that comes out of the limit, is what we call the slope at that point.



                      I guess that what is bothering you, is the notion of a limit that you learn at first-year calculus , and i felt the same when i first learn it, because at some stage of the problem we say that x approaches some value a , then at another stage we substitute a for x. But to understand what is actually going on, you have to understand the epsilon-delta definition of a limit.
                      see:https://en.wikipedia.org/wiki/(%CE%B5,_%CE%B4)-definition_of_limit






                      share|cite|improve this answer











                      $endgroup$



                      I think the problem with your thinking is that, you are assuming that there is something that is predefined by "god" which is the slope at a point that has exact value, and we are trying to reach it by the processes of taking a limit, and we can never reach that value. But that is not the case. What we do, is that we define the notion of the slope at a point , by the processes of taking a limit. And what ever the exact value that comes out of the limit, is what we call the slope at that point.



                      I guess that what is bothering you, is the notion of a limit that you learn at first-year calculus , and i felt the same when i first learn it, because at some stage of the problem we say that x approaches some value a , then at another stage we substitute a for x. But to understand what is actually going on, you have to understand the epsilon-delta definition of a limit.
                      see:https://en.wikipedia.org/wiki/(%CE%B5,_%CE%B4)-definition_of_limit







                      share|cite|improve this answer














                      share|cite|improve this answer



                      share|cite|improve this answer








                      edited Apr 19 at 20:08

























                      answered Apr 19 at 8:31









                      yousef magablehyousef magableh

                      11810




                      11810























                          1












                          $begingroup$

                          The idea is that the approximation gets better and better as $x$ gets closer to $a$, and, so to say, at $x=a$ it is no more approximative.



                          Below, a function in blue, the tangent in green and the difference in magenta. As you see, the difference decreases very quickly close to the tangency point.



                          enter image description here



                          This is not achieved with a different slope.



                          enter image description here






                          share|cite|improve this answer











                          $endgroup$


















                            1












                            $begingroup$

                            The idea is that the approximation gets better and better as $x$ gets closer to $a$, and, so to say, at $x=a$ it is no more approximative.



                            Below, a function in blue, the tangent in green and the difference in magenta. As you see, the difference decreases very quickly close to the tangency point.



                            enter image description here



                            This is not achieved with a different slope.



                            enter image description here






                            share|cite|improve this answer











                            $endgroup$
















                              1












                              1








                              1





                              $begingroup$

                              The idea is that the approximation gets better and better as $x$ gets closer to $a$, and, so to say, at $x=a$ it is no more approximative.



                              Below, a function in blue, the tangent in green and the difference in magenta. As you see, the difference decreases very quickly close to the tangency point.



                              enter image description here



                              This is not achieved with a different slope.



                              enter image description here






                              share|cite|improve this answer











                              $endgroup$



                              The idea is that the approximation gets better and better as $x$ gets closer to $a$, and, so to say, at $x=a$ it is no more approximative.



                              Below, a function in blue, the tangent in green and the difference in magenta. As you see, the difference decreases very quickly close to the tangency point.



                              enter image description here



                              This is not achieved with a different slope.



                              enter image description here







                              share|cite|improve this answer














                              share|cite|improve this answer



                              share|cite|improve this answer








                              edited Apr 19 at 8:51

























                              answered Apr 19 at 8:45









                              Yves DaoustYves Daoust

                              134k676232




                              134k676232























                                  1












                                  $begingroup$

                                  Thought I'd add something with an emphasis on geometry to the excellent answers above.



                                  Consider a point A on a circle and a point B elsewhere on the circle and draw a line between the two. The line touches the circle in two places. We have a secant line. Select another point B closer to A than the original B and draw another line. Again it intersects in two places, another secant line. Iterate bringing the new Points be ever closer to we get more and more secant lines until B is actually on A and the resulting line 1) intersects the circle at one point, and 2) is perpendicular to the radius through A. We have a tangent to our circle. We can associate a slope to the corresponding line.



                                  To many curves we can associate a tangent circle, any circle that pass through the curve at only one point. Then the slope of the tangent line to the circle is the slope of the tangent line to the curve. We have the slope of a tangent line without calculus.



                                  Consider the parabola $y=x^2$. Let's create a circle that intersects it at only one point.



                                  $(x-a)^2+(x^2-b)^2=r^2$



                                  $x^2+a^2-2ax+x^4+b^2-2bx^2=r^2$



                                  $x^4+(1-2b)x^2-2ax+(a^2+b^2-r^2)=0$



                                  Under what conditions does this have only one solution on the parabola?



                                  We can make r arbitrarily small and find an (a,b) within r of the target point, $x_0$. So we can make the constant term of the polynomial vanish reducing it to



                                  $x^3+(1-2b)x-2a=0$



                                  We want the pont who's tangency we want to find to be a solution so we divide by $(x-x_0)$.



                                  We want $x_0$ to have multiplicity 3 or the quadratic to have no real solutions. This tells us the relationship between the center of our circle, the radius and the point of tangency.






                                  share|cite|improve this answer











                                  $endgroup$


















                                    1












                                    $begingroup$

                                    Thought I'd add something with an emphasis on geometry to the excellent answers above.



                                    Consider a point A on a circle and a point B elsewhere on the circle and draw a line between the two. The line touches the circle in two places. We have a secant line. Select another point B closer to A than the original B and draw another line. Again it intersects in two places, another secant line. Iterate bringing the new Points be ever closer to we get more and more secant lines until B is actually on A and the resulting line 1) intersects the circle at one point, and 2) is perpendicular to the radius through A. We have a tangent to our circle. We can associate a slope to the corresponding line.



                                    To many curves we can associate a tangent circle, any circle that pass through the curve at only one point. Then the slope of the tangent line to the circle is the slope of the tangent line to the curve. We have the slope of a tangent line without calculus.



                                    Consider the parabola $y=x^2$. Let's create a circle that intersects it at only one point.



                                    $(x-a)^2+(x^2-b)^2=r^2$



                                    $x^2+a^2-2ax+x^4+b^2-2bx^2=r^2$



                                    $x^4+(1-2b)x^2-2ax+(a^2+b^2-r^2)=0$



                                    Under what conditions does this have only one solution on the parabola?



                                    We can make r arbitrarily small and find an (a,b) within r of the target point, $x_0$. So we can make the constant term of the polynomial vanish reducing it to



                                    $x^3+(1-2b)x-2a=0$



                                    We want the pont who's tangency we want to find to be a solution so we divide by $(x-x_0)$.



                                    We want $x_0$ to have multiplicity 3 or the quadratic to have no real solutions. This tells us the relationship between the center of our circle, the radius and the point of tangency.






                                    share|cite|improve this answer











                                    $endgroup$
















                                      1












                                      1








                                      1





                                      $begingroup$

                                      Thought I'd add something with an emphasis on geometry to the excellent answers above.



                                      Consider a point A on a circle and a point B elsewhere on the circle and draw a line between the two. The line touches the circle in two places. We have a secant line. Select another point B closer to A than the original B and draw another line. Again it intersects in two places, another secant line. Iterate bringing the new Points be ever closer to we get more and more secant lines until B is actually on A and the resulting line 1) intersects the circle at one point, and 2) is perpendicular to the radius through A. We have a tangent to our circle. We can associate a slope to the corresponding line.



                                      To many curves we can associate a tangent circle, any circle that pass through the curve at only one point. Then the slope of the tangent line to the circle is the slope of the tangent line to the curve. We have the slope of a tangent line without calculus.



                                      Consider the parabola $y=x^2$. Let's create a circle that intersects it at only one point.



                                      $(x-a)^2+(x^2-b)^2=r^2$



                                      $x^2+a^2-2ax+x^4+b^2-2bx^2=r^2$



                                      $x^4+(1-2b)x^2-2ax+(a^2+b^2-r^2)=0$



                                      Under what conditions does this have only one solution on the parabola?



                                      We can make r arbitrarily small and find an (a,b) within r of the target point, $x_0$. So we can make the constant term of the polynomial vanish reducing it to



                                      $x^3+(1-2b)x-2a=0$



                                      We want the pont who's tangency we want to find to be a solution so we divide by $(x-x_0)$.



                                      We want $x_0$ to have multiplicity 3 or the quadratic to have no real solutions. This tells us the relationship between the center of our circle, the radius and the point of tangency.






                                      share|cite|improve this answer











                                      $endgroup$



                                      Thought I'd add something with an emphasis on geometry to the excellent answers above.



                                      Consider a point A on a circle and a point B elsewhere on the circle and draw a line between the two. The line touches the circle in two places. We have a secant line. Select another point B closer to A than the original B and draw another line. Again it intersects in two places, another secant line. Iterate bringing the new Points be ever closer to we get more and more secant lines until B is actually on A and the resulting line 1) intersects the circle at one point, and 2) is perpendicular to the radius through A. We have a tangent to our circle. We can associate a slope to the corresponding line.



                                      To many curves we can associate a tangent circle, any circle that pass through the curve at only one point. Then the slope of the tangent line to the circle is the slope of the tangent line to the curve. We have the slope of a tangent line without calculus.



                                      Consider the parabola $y=x^2$. Let's create a circle that intersects it at only one point.



                                      $(x-a)^2+(x^2-b)^2=r^2$



                                      $x^2+a^2-2ax+x^4+b^2-2bx^2=r^2$



                                      $x^4+(1-2b)x^2-2ax+(a^2+b^2-r^2)=0$



                                      Under what conditions does this have only one solution on the parabola?



                                      We can make r arbitrarily small and find an (a,b) within r of the target point, $x_0$. So we can make the constant term of the polynomial vanish reducing it to



                                      $x^3+(1-2b)x-2a=0$



                                      We want the pont who's tangency we want to find to be a solution so we divide by $(x-x_0)$.



                                      We want $x_0$ to have multiplicity 3 or the quadratic to have no real solutions. This tells us the relationship between the center of our circle, the radius and the point of tangency.







                                      share|cite|improve this answer














                                      share|cite|improve this answer



                                      share|cite|improve this answer








                                      edited Apr 19 at 21:14

























                                      answered Apr 19 at 16:09









                                      TurlocTheRedTurlocTheRed

                                      1,046412




                                      1,046412























                                          0












                                          $begingroup$

                                          Well, if you have two points $(x_1,y_1)$ and $(x_2,y_2)$ in the plane, then the slope of the unique line through these points is
                                          $$ m = frac{y_2-y_1}{x_2-x_1}.$$
                                          If you consider a (differentiable) function $f:{Bbb R}rightarrow{Bbb R}$, then the slope of the line between $(x_1,f(x_1))$ and $(x_2,f(x_2))$ is
                                          $$ m = frac{f(x_2)-f(x_1)}{x_2-x_1}.$$
                                          If you take the points $x_0$ and $x$ at the x-axis, then consider the quotient
                                          $$frac{f(x)-f(x_0)}{x-x_0}$$
                                          when $x$ moves to $x_0$: $xrightarrow x_0$.
                                          Then this quotient becomes (in the limit) the slope of the tangent line of $f$ at the point $x_0$. This is particularly easy to understand if you draw a picture.



                                          Denote the slope in the limit as $f'(x_0)$. Then the above quotient gives
                                          $$f'(x_0) = frac{f(x)-f(x_0)}{x-x_0}$$
                                          and so the equation of the tangent line
                                          $$f(x) = f'(x_0)(x-x_0) + f(x_0).$$






                                          share|cite|improve this answer











                                          $endgroup$













                                          • $begingroup$
                                            Since $x_0$ is an approximate , Then why we get the exact value of the slope ?
                                            $endgroup$
                                            – Mohammad Alshareef
                                            Apr 19 at 7:25








                                          • 1




                                            $begingroup$
                                            @Mohammad Alshareef: The "exact value of the slope at a given point" is defined using a limit, or do you have another way of defining it? Your question seems to me like asking why we use three letters to write the word "big". (Answer: Because the word "big" is defined by making use of three letters, or something to this effect.)
                                            $endgroup$
                                            – Dave L. Renfro
                                            Apr 19 at 7:42










                                          • $begingroup$
                                            @DaveL.Renfro : Does $f(xto a) = f(x=a)$ ?
                                            $endgroup$
                                            – Mohammad Alshareef
                                            Apr 19 at 8:06












                                          • $begingroup$
                                            Yes, but that's because (i) the function $g(x) = x$ is continuous [note that your two inputs are "$x rightarrow a$" and "$a$"; these are equal because of (i)] and (ii) $f$ is (presumably) a function [$f$ a function implies that whenever $c=d,$ then we have $f(c) = f(d)].$ Perhaps you wanted to say "limit as $x rightarrow a$ of $f(x)$" is equal to "$f(a)$"? (This is not true, however, unless $f$ is continuous at $x=0.)$
                                            $endgroup$
                                            – Dave L. Renfro
                                            Apr 19 at 8:13












                                          • $begingroup$
                                            Actually ,This explain everything ,But I am not satisfied , Since $(xto a) + h = a$ , $h ≠ 0$ , Then $f(xto a)$ should not be equal to $f(a)$.
                                            $endgroup$
                                            – Mohammad Alshareef
                                            Apr 19 at 8:24


















                                          0












                                          $begingroup$

                                          Well, if you have two points $(x_1,y_1)$ and $(x_2,y_2)$ in the plane, then the slope of the unique line through these points is
                                          $$ m = frac{y_2-y_1}{x_2-x_1}.$$
                                          If you consider a (differentiable) function $f:{Bbb R}rightarrow{Bbb R}$, then the slope of the line between $(x_1,f(x_1))$ and $(x_2,f(x_2))$ is
                                          $$ m = frac{f(x_2)-f(x_1)}{x_2-x_1}.$$
                                          If you take the points $x_0$ and $x$ at the x-axis, then consider the quotient
                                          $$frac{f(x)-f(x_0)}{x-x_0}$$
                                          when $x$ moves to $x_0$: $xrightarrow x_0$.
                                          Then this quotient becomes (in the limit) the slope of the tangent line of $f$ at the point $x_0$. This is particularly easy to understand if you draw a picture.



                                          Denote the slope in the limit as $f'(x_0)$. Then the above quotient gives
                                          $$f'(x_0) = frac{f(x)-f(x_0)}{x-x_0}$$
                                          and so the equation of the tangent line
                                          $$f(x) = f'(x_0)(x-x_0) + f(x_0).$$






                                          share|cite|improve this answer











                                          $endgroup$













                                          • $begingroup$
                                            Since $x_0$ is an approximate , Then why we get the exact value of the slope ?
                                            $endgroup$
                                            – Mohammad Alshareef
                                            Apr 19 at 7:25








                                          • 1




                                            $begingroup$
                                            @Mohammad Alshareef: The "exact value of the slope at a given point" is defined using a limit, or do you have another way of defining it? Your question seems to me like asking why we use three letters to write the word "big". (Answer: Because the word "big" is defined by making use of three letters, or something to this effect.)
                                            $endgroup$
                                            – Dave L. Renfro
                                            Apr 19 at 7:42










                                          • $begingroup$
                                            @DaveL.Renfro : Does $f(xto a) = f(x=a)$ ?
                                            $endgroup$
                                            – Mohammad Alshareef
                                            Apr 19 at 8:06












                                          • $begingroup$
                                            Yes, but that's because (i) the function $g(x) = x$ is continuous [note that your two inputs are "$x rightarrow a$" and "$a$"; these are equal because of (i)] and (ii) $f$ is (presumably) a function [$f$ a function implies that whenever $c=d,$ then we have $f(c) = f(d)].$ Perhaps you wanted to say "limit as $x rightarrow a$ of $f(x)$" is equal to "$f(a)$"? (This is not true, however, unless $f$ is continuous at $x=0.)$
                                            $endgroup$
                                            – Dave L. Renfro
                                            Apr 19 at 8:13












                                          • $begingroup$
                                            Actually ,This explain everything ,But I am not satisfied , Since $(xto a) + h = a$ , $h ≠ 0$ , Then $f(xto a)$ should not be equal to $f(a)$.
                                            $endgroup$
                                            – Mohammad Alshareef
                                            Apr 19 at 8:24
















                                          0












                                          0








                                          0





                                          $begingroup$

                                          Well, if you have two points $(x_1,y_1)$ and $(x_2,y_2)$ in the plane, then the slope of the unique line through these points is
                                          $$ m = frac{y_2-y_1}{x_2-x_1}.$$
                                          If you consider a (differentiable) function $f:{Bbb R}rightarrow{Bbb R}$, then the slope of the line between $(x_1,f(x_1))$ and $(x_2,f(x_2))$ is
                                          $$ m = frac{f(x_2)-f(x_1)}{x_2-x_1}.$$
                                          If you take the points $x_0$ and $x$ at the x-axis, then consider the quotient
                                          $$frac{f(x)-f(x_0)}{x-x_0}$$
                                          when $x$ moves to $x_0$: $xrightarrow x_0$.
                                          Then this quotient becomes (in the limit) the slope of the tangent line of $f$ at the point $x_0$. This is particularly easy to understand if you draw a picture.



                                          Denote the slope in the limit as $f'(x_0)$. Then the above quotient gives
                                          $$f'(x_0) = frac{f(x)-f(x_0)}{x-x_0}$$
                                          and so the equation of the tangent line
                                          $$f(x) = f'(x_0)(x-x_0) + f(x_0).$$






                                          share|cite|improve this answer











                                          $endgroup$



                                          Well, if you have two points $(x_1,y_1)$ and $(x_2,y_2)$ in the plane, then the slope of the unique line through these points is
                                          $$ m = frac{y_2-y_1}{x_2-x_1}.$$
                                          If you consider a (differentiable) function $f:{Bbb R}rightarrow{Bbb R}$, then the slope of the line between $(x_1,f(x_1))$ and $(x_2,f(x_2))$ is
                                          $$ m = frac{f(x_2)-f(x_1)}{x_2-x_1}.$$
                                          If you take the points $x_0$ and $x$ at the x-axis, then consider the quotient
                                          $$frac{f(x)-f(x_0)}{x-x_0}$$
                                          when $x$ moves to $x_0$: $xrightarrow x_0$.
                                          Then this quotient becomes (in the limit) the slope of the tangent line of $f$ at the point $x_0$. This is particularly easy to understand if you draw a picture.



                                          Denote the slope in the limit as $f'(x_0)$. Then the above quotient gives
                                          $$f'(x_0) = frac{f(x)-f(x_0)}{x-x_0}$$
                                          and so the equation of the tangent line
                                          $$f(x) = f'(x_0)(x-x_0) + f(x_0).$$







                                          share|cite|improve this answer














                                          share|cite|improve this answer



                                          share|cite|improve this answer








                                          edited Apr 19 at 7:49

























                                          answered Apr 19 at 7:13









                                          WuestenfuxWuestenfux

                                          5,6211513




                                          5,6211513












                                          • $begingroup$
                                            Since $x_0$ is an approximate , Then why we get the exact value of the slope ?
                                            $endgroup$
                                            – Mohammad Alshareef
                                            Apr 19 at 7:25








                                          • 1




                                            $begingroup$
                                            @Mohammad Alshareef: The "exact value of the slope at a given point" is defined using a limit, or do you have another way of defining it? Your question seems to me like asking why we use three letters to write the word "big". (Answer: Because the word "big" is defined by making use of three letters, or something to this effect.)
                                            $endgroup$
                                            – Dave L. Renfro
                                            Apr 19 at 7:42










                                          • $begingroup$
                                            @DaveL.Renfro : Does $f(xto a) = f(x=a)$ ?
                                            $endgroup$
                                            – Mohammad Alshareef
                                            Apr 19 at 8:06












                                          • $begingroup$
                                            Yes, but that's because (i) the function $g(x) = x$ is continuous [note that your two inputs are "$x rightarrow a$" and "$a$"; these are equal because of (i)] and (ii) $f$ is (presumably) a function [$f$ a function implies that whenever $c=d,$ then we have $f(c) = f(d)].$ Perhaps you wanted to say "limit as $x rightarrow a$ of $f(x)$" is equal to "$f(a)$"? (This is not true, however, unless $f$ is continuous at $x=0.)$
                                            $endgroup$
                                            – Dave L. Renfro
                                            Apr 19 at 8:13












                                          • $begingroup$
                                            Actually ,This explain everything ,But I am not satisfied , Since $(xto a) + h = a$ , $h ≠ 0$ , Then $f(xto a)$ should not be equal to $f(a)$.
                                            $endgroup$
                                            – Mohammad Alshareef
                                            Apr 19 at 8:24




















                                          • $begingroup$
                                            Since $x_0$ is an approximate , Then why we get the exact value of the slope ?
                                            $endgroup$
                                            – Mohammad Alshareef
                                            Apr 19 at 7:25








                                          • 1




                                            $begingroup$
                                            @Mohammad Alshareef: The "exact value of the slope at a given point" is defined using a limit, or do you have another way of defining it? Your question seems to me like asking why we use three letters to write the word "big". (Answer: Because the word "big" is defined by making use of three letters, or something to this effect.)
                                            $endgroup$
                                            – Dave L. Renfro
                                            Apr 19 at 7:42










                                          • $begingroup$
                                            @DaveL.Renfro : Does $f(xto a) = f(x=a)$ ?
                                            $endgroup$
                                            – Mohammad Alshareef
                                            Apr 19 at 8:06












                                          • $begingroup$
                                            Yes, but that's because (i) the function $g(x) = x$ is continuous [note that your two inputs are "$x rightarrow a$" and "$a$"; these are equal because of (i)] and (ii) $f$ is (presumably) a function [$f$ a function implies that whenever $c=d,$ then we have $f(c) = f(d)].$ Perhaps you wanted to say "limit as $x rightarrow a$ of $f(x)$" is equal to "$f(a)$"? (This is not true, however, unless $f$ is continuous at $x=0.)$
                                            $endgroup$
                                            – Dave L. Renfro
                                            Apr 19 at 8:13












                                          • $begingroup$
                                            Actually ,This explain everything ,But I am not satisfied , Since $(xto a) + h = a$ , $h ≠ 0$ , Then $f(xto a)$ should not be equal to $f(a)$.
                                            $endgroup$
                                            – Mohammad Alshareef
                                            Apr 19 at 8:24


















                                          $begingroup$
                                          Since $x_0$ is an approximate , Then why we get the exact value of the slope ?
                                          $endgroup$
                                          – Mohammad Alshareef
                                          Apr 19 at 7:25






                                          $begingroup$
                                          Since $x_0$ is an approximate , Then why we get the exact value of the slope ?
                                          $endgroup$
                                          – Mohammad Alshareef
                                          Apr 19 at 7:25






                                          1




                                          1




                                          $begingroup$
                                          @Mohammad Alshareef: The "exact value of the slope at a given point" is defined using a limit, or do you have another way of defining it? Your question seems to me like asking why we use three letters to write the word "big". (Answer: Because the word "big" is defined by making use of three letters, or something to this effect.)
                                          $endgroup$
                                          – Dave L. Renfro
                                          Apr 19 at 7:42




                                          $begingroup$
                                          @Mohammad Alshareef: The "exact value of the slope at a given point" is defined using a limit, or do you have another way of defining it? Your question seems to me like asking why we use three letters to write the word "big". (Answer: Because the word "big" is defined by making use of three letters, or something to this effect.)
                                          $endgroup$
                                          – Dave L. Renfro
                                          Apr 19 at 7:42












                                          $begingroup$
                                          @DaveL.Renfro : Does $f(xto a) = f(x=a)$ ?
                                          $endgroup$
                                          – Mohammad Alshareef
                                          Apr 19 at 8:06






                                          $begingroup$
                                          @DaveL.Renfro : Does $f(xto a) = f(x=a)$ ?
                                          $endgroup$
                                          – Mohammad Alshareef
                                          Apr 19 at 8:06














                                          $begingroup$
                                          Yes, but that's because (i) the function $g(x) = x$ is continuous [note that your two inputs are "$x rightarrow a$" and "$a$"; these are equal because of (i)] and (ii) $f$ is (presumably) a function [$f$ a function implies that whenever $c=d,$ then we have $f(c) = f(d)].$ Perhaps you wanted to say "limit as $x rightarrow a$ of $f(x)$" is equal to "$f(a)$"? (This is not true, however, unless $f$ is continuous at $x=0.)$
                                          $endgroup$
                                          – Dave L. Renfro
                                          Apr 19 at 8:13






                                          $begingroup$
                                          Yes, but that's because (i) the function $g(x) = x$ is continuous [note that your two inputs are "$x rightarrow a$" and "$a$"; these are equal because of (i)] and (ii) $f$ is (presumably) a function [$f$ a function implies that whenever $c=d,$ then we have $f(c) = f(d)].$ Perhaps you wanted to say "limit as $x rightarrow a$ of $f(x)$" is equal to "$f(a)$"? (This is not true, however, unless $f$ is continuous at $x=0.)$
                                          $endgroup$
                                          – Dave L. Renfro
                                          Apr 19 at 8:13














                                          $begingroup$
                                          Actually ,This explain everything ,But I am not satisfied , Since $(xto a) + h = a$ , $h ≠ 0$ , Then $f(xto a)$ should not be equal to $f(a)$.
                                          $endgroup$
                                          – Mohammad Alshareef
                                          Apr 19 at 8:24






                                          $begingroup$
                                          Actually ,This explain everything ,But I am not satisfied , Since $(xto a) + h = a$ , $h ≠ 0$ , Then $f(xto a)$ should not be equal to $f(a)$.
                                          $endgroup$
                                          – Mohammad Alshareef
                                          Apr 19 at 8:24




















                                          draft saved

                                          draft discarded




















































                                          Thanks for contributing an answer to Mathematics Stack Exchange!


                                          • Please be sure to answer the question. Provide details and share your research!

                                          But avoid



                                          • Asking for help, clarification, or responding to other answers.

                                          • Making statements based on opinion; back them up with references or personal experience.


                                          Use MathJax to format equations. MathJax reference.


                                          To learn more, see our tips on writing great answers.




                                          draft saved


                                          draft discarded














                                          StackExchange.ready(
                                          function () {
                                          StackExchange.openid.initPostLogin('.new-post-login', 'https%3a%2f%2fmath.stackexchange.com%2fquestions%2f3193260%2fwhy-limits-give-us-the-exact-value-of-the-slope-of-the-tangent-line%23new-answer', 'question_page');
                                          }
                                          );

                                          Post as a guest















                                          Required, but never shown





















































                                          Required, but never shown














                                          Required, but never shown












                                          Required, but never shown







                                          Required, but never shown

































                                          Required, but never shown














                                          Required, but never shown












                                          Required, but never shown







                                          Required, but never shown







                                          Popular posts from this blog

                                          Plaza Victoria

                                          In PowerPoint, is there a keyboard shortcut for bulleted / numbered list?

                                          How to put 3 figures in Latex with 2 figures side by side and 1 below these side by side images but in...